www.vorkurse.de
Ein Projekt von vorhilfe.de
Die Online-Kurse der Vorhilfe

E-Learning leicht gemacht.
Hallo Gast!einloggen | registrieren ]
Startseite · Mitglieder · Teams · Forum · Wissen · Kurse · Impressum
Forenbaum
^ Forenbaum
Status Mathe-Vorkurse
  Status Organisatorisches
  Status Schule
    Status Wiederholung Algebra
    Status Einführung Analysis
    Status Einführung Analytisc
    Status VK 21: Mathematik 6.
    Status VK 37: Kurvendiskussionen
    Status VK Abivorbereitungen
  Status Universität
    Status Lerngruppe LinAlg
    Status VK 13 Analysis I FH
    Status Algebra 2006
    Status VK 22: Algebra 2007
    Status GruMiHH 06
    Status VK 58: Algebra 1
    Status VK 59: Lineare Algebra
    Status VK 60: Analysis
    Status Wahrscheinlichkeitst

Gezeigt werden alle Foren bis zur Tiefe 2

Navigation
 Startseite...
 Neuerdings beta neu
 Forum...
 vorwissen...
 vorkurse...
 Werkzeuge...
 Nachhilfevermittlung beta...
 Online-Spiele beta
 Suchen
 Verein...
 Impressum
Das Projekt
Server und Internetanbindung werden durch Spenden finanziert.
Organisiert wird das Projekt von unserem Koordinatorenteam.
Hunderte Mitglieder helfen ehrenamtlich in unseren moderierten Foren.
Anbieter der Seite ist der gemeinnützige Verein "Vorhilfe.de e.V.".
Partnerseiten
Weitere Fächer:

Open Source FunktionenplotterFunkyPlot: Kostenloser und quelloffener Funktionenplotter für Linux und andere Betriebssysteme
Forum "Uni-Numerik" - Basispolynome
Basispolynome < Numerik < Hochschule < Mathe < Vorhilfe
Ansicht: [ geschachtelt ] | ^ Forum "Uni-Numerik"  | ^^ Alle Foren  | ^ Forenbaum  | Materialien

Basispolynome: Frage (beantwortet)
Status: (Frage) beantwortet Status 
Datum: 12:11 Di 18.08.2009
Autor: tynia

Aufgabe
Zeigen Sie, dass die folgenden Funktionensysteme jeweils eine Basis im Vektorraum der Polynome bis zum Grad 3 bilden:

a) [mm] B_{1}=(1,x,x^{2},x^{3}) [/mm]

b) [mm] B_{2}=(1,(x+1),(x+1)(x-1),(x+1)(x-1)(x-4)) [/mm]

c) [mm] B_{3}=((x-1)(x-2)(x-4),(x+1)(x-2)(x-4),(x+1)(x-1)(x-4),(x+1)(x-1)(x-2)) [/mm]



Bemerkung: [mm] B_{2} [/mm] sind die Newton’schen, [mm] B_{3} [/mm] die Lagrange’schen Basispolynome zu den Stützstellen -1, 1, 2, 4.

Hallo. Muss obige Aufgabe lösen und weiß irgendwie nicht, wie ich das beginnen soll. Kann mir jemand hier vielleicht einen Ansatz geben? Ich stehe echt auf dem Schlauch. Bin über jeden Tipp dankbar.

LG

        
Bezug
Basispolynome: Antwort
Status: (Antwort) fertig Status 
Datum: 12:18 Di 18.08.2009
Autor: angela.h.b.


> Zeigen Sie, dass die folgenden Funktionensysteme jeweils
> eine Basis im Vektorraum der Polynome bis zum Grad 3
> bilden:
>  
> a) [mm]B_{1}=(1,x,x^{2},x^{3})[/mm]
>  
> b) [mm]B_{2}=(1,(x+1),(x+1)(x-1),(x+1)(x-1)(x-4))[/mm]
>  
> c)
> [mm]B_{3}=((x-1)(x-2)(x-4),(x+1)(x-2)(x-4),(x+1)(x-1)(x-4),(x+1)(x-1)(x-2))[/mm]
>  
>
>
> Bemerkung: [mm]B_{2}[/mm] sind die Newton’schen, [mm]B_{3}[/mm] die
> Lagrange’schen Basispolynome zu den Stützstellen -1, 1,
> 2, 4.
>  Hallo. Muss obige Aufgabe lösen und weiß irgendwie
> nicht, wie ich das beginnen soll. Kann mir jemand hier
> vielleicht einen Ansatz geben? Ich stehe echt auf dem
> Schlauch. Bin über jeden Tipp dankbar.

Hallo,

zeig' bei a) , daß [mm] B_1 [/mm] ein Erzeugendensystem ist und linear unabhängig.

Damit weißt Du dann, daß der besagte Polynomraum die Dimension 4 hat.

Wenn Du das weißt, kannst Du Dich bei den darauffolgenden Teilaufgaben auf die lineare unabhängigkeit beschränken, denn es sind ja jedesmal 4 Polynome.

Gruß v. Angela

Bezug
                
Bezug
Basispolynome: Frage (beantwortet)
Status: (Frage) beantwortet Status 
Datum: 12:34 Di 18.08.2009
Autor: tynia

Zu a)

[mm] B_{1}=(1,x,x^{2},x^{3}) [/mm]

[mm] B_{11}= \begin{pmatrix} 1 \\ 0 \\ 0 \\ 0 \end{pmatrix} B_{12}= \begin{pmatrix} 0 \\ x \\ 0 \\ 0 \end{pmatrix} B_{13}= \begin{pmatrix} 0 \\ 0 \\ x^{2} \\ 0 \end{pmatrix} B_{14}= \begin{pmatrix} 0 \\ 0 \\ 0 \\ x^{3} \end{pmatrix} [/mm]

Lineare Unabhängigkeit bzw. Abhängigkeit

Vektoren sind genau dann linear unabhängig, wenn man den Nullvektor mit Hilfe dieser Vektoren nur mit Hilfe der trivialen Lösung erzeugen kann, sprich:

Zu zeigen ist also, dass aus

[mm] \summe_{i=1}^{n} \lambda_{i}B_{1i} [/mm] = 0 folgt, dass [mm] \lambda_{i}=0 {\forall i=(1,...,n)} [/mm]

Für meine Lösung bedeutet das doch, dass die Vektoren linear unabhängig sind, da aus [mm] \summe_{i=1}^{4} \lambda_{i}B_{1i} [/mm] = 0 folgt, dass [mm] \lambda_{i}=0 {\forall i=(1,...,4)} [/mm]

Reicht das wirklich so, oder muss ich da irgendwas berechnen?

Bezug
                        
Bezug
Basispolynome: Antwort
Status: (Antwort) fertig Status 
Datum: 12:56 Di 18.08.2009
Autor: fred97


> Zu a)
>  
> [mm]B_{1}=(1,x,x^{2},x^{3})[/mm]
>  
> [mm]B_{11}= \begin{pmatrix} 1 \\ 0 \\ 0 \\ 0 \end{pmatrix} B_{12}= \begin{pmatrix} 0 \\ x \\ 0 \\ 0 \end{pmatrix} B_{13}= \begin{pmatrix} 0 \\ 0 \\ x^{2} \\ 0 \end{pmatrix} B_{14}= \begin{pmatrix} 0 \\ 0 \\ 0 \\ x^{3} \end{pmatrix}[/mm]


Was soll denn das ????

Zeige: aus

               [mm] $\lambda_0 +\lambda_1 [/mm] x + [mm] \lambda_2 x^2 [/mm] + [mm] \lambda_3 x^3 [/mm] = 0$  für jedes x [mm] \in \IR [/mm]

folgt: [mm] $\lambda_0 [/mm] = ... = [mm] \lambda_3 [/mm] = 0$

FRED



>  
> Lineare Unabhängigkeit bzw. Abhängigkeit
>  
> Vektoren sind genau dann linear unabhängig, wenn man den
> Nullvektor mit Hilfe dieser Vektoren nur mit Hilfe der
> trivialen Lösung erzeugen kann, sprich:
>  
> Zu zeigen ist also, dass aus
>  
> [mm]\summe_{i=1}^{n} \lambda_{i}B_{1i}[/mm] = 0 folgt, dass
> [mm]\lambda_{i}=0 {\forall i=(1,...,n)}[/mm]
>  
> Für meine Lösung bedeutet das doch, dass die Vektoren
> linear unabhängig sind, da aus [mm]\summe_{i=1}^{4} \lambda_{i}B_{1i}[/mm]
> = 0 folgt, dass [mm]\lambda_{i}=0 {\forall i=(1,...,4)}[/mm]
>  
> Reicht das wirklich so, oder muss ich da irgendwas
> berechnen?


Bezug
                                
Bezug
Basispolynome: Frage (beantwortet)
Status: (Frage) beantwortet Status 
Datum: 13:01 Di 18.08.2009
Autor: tynia

Ist das falsch, wie ich es aufgeschrieben habe?

Und wie soll ich zeigen, dass aus [mm] \lambda_0 +\lambda_1 [/mm] x + [mm] \lambda_2 x^2 [/mm] + [mm] \lambda_3 x^3 [/mm] = 0  für jedes x [mm] \in \IR [/mm] folgt: [mm] \lambda_0 [/mm] = ... = [mm] \lambda_3 [/mm] = 0 ?

Das sieht man doch, dass [mm] \lambda_0 [/mm] = ... = [mm] \lambda_3 [/mm] = 0 sein muss. Ich wüsste jetzt nicht wie ich das zeigen soll.





Bezug
                                        
Bezug
Basispolynome: Antwort
Status: (Antwort) fertig Status 
Datum: 13:19 Di 18.08.2009
Autor: angela.h.b.


> Ist das falsch, wie ich es aufgeschrieben habe?

Hallo,

ja, das war sogar ganz grottenfalsch, und ich bekomme den furchtbaren Verdacht, daß Du bisher überhaupt nicht verstanden hast, was ein Vektor ist.
Dir ist klar, daß "Vektor" nicht automatisch etwas mit solchen Spalten zu tun hat, wie Du sie aufgeschrieben hast?

Ein Vektor ist ein Element eines Vektorraumes. Nicht mehr, nicht weniger.

Dein Vektorraum besteht nun aus Polynomen vom Höchstgrad 3. Also sind die Vektoren hier Polynome vom Höchstgrad 3, und entsprechend besteht die basis dieses VRes aus Polynomen vom Höchstgrad 3.
Der Nullvektor ist in diesem Fall das Nullpolynom, das war dran, als Ihr gezeigt habt, daß die Polynome einen VR bilden.

>  
> Und wie soll ich zeigen, dass aus [mm]\lambda_0 +\lambda_1[/mm] x +
> [mm]\lambda_2 x^2[/mm] + [mm]\lambda_3 x^3[/mm] = 0  für jedes x [mm]\in \IR[/mm]
> folgt: [mm]\lambda_0[/mm] = ... = [mm]\lambda_3[/mm] = 0 ?
>  
> Das sieht man doch, dass [mm]\lambda_0[/mm] = ... = [mm]\lambda_3[/mm] = 0
> sein muss. Ich wüsste jetzt nicht wie ich das zeigen
> soll.

Naja, zeigen ist vielleicht zuviel gesagt: Du hast auf der einen Seite das Polynom [mm]\lambda_0 +\lambda_1[/mm] x +  [mm]\lambda_2 x^2[/mm] + [mm]\lambda_3 x^3[/mm] , auf der anderen Seite das Nullpolynom. Wann stimmen zwei Polynome überein?  Damit hast Du's dann...


Bei den anderen drei Teilaufgaben könntest Du auch so vorgehen, daß Du die angebenen Polynome als Koordinatenvektoren bzgl der Basis [mm] B_1 [/mm] schreibst. Damit hättest Du dann Deine Spalten. Ich würde Dir aber raten, noch zwei der Aufgaben ohne Koordinatenvektoren zu lösen, und vielleicht die letzte mit. (Jedenfalls, wenn es Dir nicht auf Geschwindigkeit ankommt, sondern Du ein bißchen etwas verstehen möchtest.)

Gruß v. Angela

Bezug
                                                
Bezug
Basispolynome: Frage (beantwortet)
Status: (Frage) beantwortet Status 
Datum: 13:36 Di 18.08.2009
Autor: tynia

Du hast recht, ich verstehe hier von dem ganzen nicht wirklich viel. Aber ich gebe mein bestes.

Nochmal zu meiner Aufgabe:

Zwei Polynome sind genau dann gleich, wenn sie in allen Koeffizienten übereinstimmen, d.h. da alle Koeffizienten [mm] a_{i} [/mm] beim Nullpolynom 0 sind, müssen alle [mm] \lambda_{i} [/mm] = 0 sein.

Du hast recht, ich mache das wohl ohne Koordinatenvektoren. ich habe das nur so gemacht, weil ich in meinen Unterlagen eine Lösung hatte, die so gemacht wurde.

Ich mache jetzt mal die anderen Aufgaben auch und poste die gleich. Viellleicht kannst du mir sagen, ob das dann so richtig ist.




Bezug
                                                        
Bezug
Basispolynome: Antwort
Status: (Antwort) fertig Status 
Datum: 13:45 Di 18.08.2009
Autor: angela.h.b.


> Zwei Polynome sind genau dann gleich, wenn sie in allen
> Koeffizienten übereinstimmen, d.h. da alle Koeffizienten
> [mm]a_{i}[/mm] beim Nullpolynom 0 sind, müssen alle [mm]\lambda_{i}[/mm] = 0
> sein.

Hallo,

ja, das ist eine Begründung, die reicht.

Gruß v. Angela

Bezug
        
Bezug
Basispolynome: Frage (beantwortet)
Status: (Frage) beantwortet Status 
Datum: 14:06 Di 18.08.2009
Autor: tynia

Ich habe jetzt mal b) gemacht und würde nun gerne wissen, ob das stimmt.


Newtonsches Basispolynom

p(x)= [mm] c_{0}+c_{1}(x-x_{0})+c_{2}(x-x_{0})(x-x_{1})+...+ c_{n}(x-x_{0}) \ldots (x-x_{n-1}) [/mm] soll 0 sein

Ich habe ja die Stützstellen gegeben und setze jetzt einfach ein:

p(x)= [mm] c_{0}+c_{1}(x+1)+c_{2}(x+1)(x-1)+ c_{3}(x+1)(x-1)(x-2)=0 [/mm]

= [mm] c_{0}+c_{1}x+c_{1}+c_{2}x^{2}-c_{2}+c_{3}x^{3}-2c_{3}x^{2}-c_{3}x+2c_{3}=0 [/mm]

= [mm] (c_{0}+c_{1}-c_{2}+2c_{3})+(c_{1}-c_{3})x+(c_{2}-2c_{3})x^{2}+c_{3}x^{3}=0 [/mm]

[mm] \Rightarrow c_{i}=0 [/mm]  für alle  i

Bezug
                
Bezug
Basispolynome: Antwort
Status: (Antwort) fertig Status 
Datum: 14:20 Di 18.08.2009
Autor: MathePower

Hallo tynia,

> Ich habe jetzt mal b) gemacht und würde nun gerne wissen,
> ob das stimmt.
>  
>
> Newtonsches Basispolynom
>  
> p(x)= [mm]c_{0}+c_{1}(x-x_{0})+c_{2}(x-x_{0})(x-x_{1})+...+ c_{n}(x-x_{0}) \ldots (x-x_{n-1})[/mm]
> soll 0 sein
>  
> Ich habe ja die Stützstellen gegeben und setze jetzt
> einfach ein:
>  
> p(x)= [mm]c_{0}+c_{1}(x+1)+c_{2}(x+1)(x-1)+ c_{3}(x+1)(x-1)(x-2)=0[/mm]
>  
> =
> [mm]c_{0}+c_{1}x+c_{1}+c_{2}x^{2}-c_{2}+c_{3}x^{3}-2c_{3}x^{2}-c_{3}x+2c_{3}=0[/mm]
>  
> =
> [mm](c_{0}+c_{1}-c_{2}+2c_{3})+(c_{1}-c_{3})x+(c_{2}-2c_{3})x^{2}+c_{3}x^{3}=0[/mm]
>  
> [mm]\Rightarrow c_{i}=0[/mm]  für alle  i


Ja, das stimmt. [ok]


Gruss
MathePower

Bezug
                        
Bezug
Basispolynome: Mitteilung
Status: (Mitteilung) Reaktion unnötig Status 
Datum: 14:24 Di 18.08.2009
Autor: tynia

Juhu :-) Langsam gehts voran. Danke schön

Bezug
                        
Bezug
Basispolynome: Frage (beantwortet)
Status: (Frage) beantwortet Status 
Datum: 16:12 Di 18.08.2009
Autor: tynia

So habe ich das ja jetzt gelöst:

Newtonsches Basispolynom

p(x)= [mm][mm] c_{0}+c_{1}(x-x_{0})+c_{2}(x-x_{0})(x-x_{1})+...+ c_{n}(x-x_{0}) \ldots (x-x_{n-1})soll [/mm] 0 sein

Ich habe ja die Stützstellen gegeben und setze jetzt einfach ein:

p(x)= [mm] c_{0}+c_{1}(x+1)+c_{2}(x+1)(x-1)+ c_{3}(x+1)(x-1)(x-2)=0 [/mm]

> > =

[mm] c_{0}+c_{1}x+c_{1}+c_{2}x^{2}-c_{2}+c_{3}x^{3}-2c_{3}x^{2}-c_{3}x+2c_{3}=0 [/mm]
= [mm] (c_{0}+c_{1}-c_{2}+2c_{3})+(c_{1}-c_{3})x+(c_{2}-2c_{3})x^{2}+c_{3}x^{3}=0 [/mm]

[mm] \Rightarrow c_{i}=0 [/mm] für alle  i

Diese Aufgabe wurde vom Tutor korriegiert und es gab volle Punktzahl.
Jetzt habe ich von einer Freundin aber noch ne andere Lösung, die auch volle Punktzahl bekommen hat. Sie hat das aber mit den Koordinatenvektoren gemacht.

Verstehe ich irgendwie nicht. Vielleicht kann mir das einer von euch erklären. Ich poste mal die andere Lösung:

[mm] B_{2}=(1,(x+1),(x+1)(x-1),(x+1)(x-1)(x-4)) [/mm]

[mm] B_{21}=\begin{pmatrix} 1 \\ 0 \\ 0 \\ 0 \end{pmatrix} B_{22}=\begin{pmatrix} 1 \\ x \\ 0 \\ 0 \end{pmatrix} B_{23}=\begin{pmatrix} -1 \\ 0 \\ x^{2} \\ 0 \end{pmatrix} B_{24}=\begin{pmatrix} 4 \\ -x \\ -4x^{2} \\ x^{3} \end{pmatrix} [/mm]

Sind linear unabhängig [mm] \to \summe_{i=1}^{} \lambda_{i}*B_{2i}=0 [/mm] falls [mm] \lambda_{i}=0 [/mm]

Das ist alles zu der Aufgabe. Ich bedanke mich schonmal im Voraus.

LG



Bezug
                                
Bezug
Basispolynome: Antwort
Status: (Antwort) fertig Status 
Datum: 16:27 Di 18.08.2009
Autor: angela.h.b.


> Verstehe ich irgendwie nicht.

Hallo,

das kann man auch nicht verstehen, weil es falsch ist.

> [mm]B_{2}=(b_1:=1,b_2:=(x+1),b_3:=(x+1)(x-1),b_4:=(x+1)(x-1)(x-4))[/mm].

Man kann nun die 4 Vektoren [mm] b_i [/mm] als Koordinatenvektoren bzgl der Standardbasis [mm] B_1:=(1,x,x^2,X^3) [/mm]  Deines Raumes schreiben:

[mm] b_1=1*1+1*x+0*x^2+0*x^3=\vektor{1\\1\\0\\0}_{(B_1)} [/mm]

[mm] b_2=1*1+0*x+0*x^2+0*x^3=\vektor{1\\0\\0\\0}_{(B_1)} [/mm]

[mm] b_3=-1*1+0*x+1*x^2+0*x^3=\vektor{-1\\0\\1\\0}_{(B_1)} [/mm]

[mm] b_4= [/mm] ...   (entsprechend- hab ich keine Lust dazu)

Nun stellt man fest: die 4 Koordinatenvektoren sind offensichtlich linear unabhängig, also sind die [mm] b_i [/mm] linear unabhängig.

Gruß v. Angela

Bezug
                                        
Bezug
Basispolynome: Mitteilung
Status: (Mitteilung) Reaktion unnötig Status 
Datum: 16:34 Di 18.08.2009
Autor: tynia

Ok. Dann lassen wir das einfach :-)

ich mache das einfach so, wie ich es verstanden habe und auch hier gepostet habe. Danke nochmal

Bezug
                                                
Bezug
Basispolynome: Mitteilung
Status: (Mitteilung) Reaktion unnötig Status 
Datum: 07:54 Mi 19.08.2009
Autor: angela.h.b.


> Ok. Dann lassen wir das einfach :-)

Hallo,

aber Dir ist klar, daß ich es Dir jetzt richtig vorgemacht habe?
Wenn man mit Koordinatenvektoren umgehen kann, ist das ja nicht unpraktisch.

Gruß v. Angela

Bezug
Ansicht: [ geschachtelt ] | ^ Forum "Uni-Numerik"  | ^^ Alle Foren  | ^ Forenbaum  | Materialien


^ Seitenanfang ^
www.vorkurse.de
[ Startseite | Mitglieder | Teams | Forum | Wissen | Kurse | Impressum ]